Difference between revisions of "2022 AMC 12B Problems/Problem 22"

(Added problem, add solution in one sec)
 
(Redirected page to 2022 AMC 10B Problems/Problem 23)
(Tag: New redirect)
 
(32 intermediate revisions by 5 users not shown)
Line 1: Line 1:
==Problem==
+
#redirect [[2022 AMC 10B Problems/Problem 23]]
Ant Amelia starts on the number line at <math>0</math> and crawls in the following manner. For <math>n=1,2,3,</math> Amelia chooses a time duration <math>t_n</math> and an increment <math>x_n</math> independently and uniformly at random from the interval <math>(0,1).</math> During the <math>n</math>th step of the process, Amelia moves <math>x_n</math> units in the positive direction, using up <math>t_n</math> minutes. If the total elapsed time has exceeded <math>1</math> minute during the <math>n</math>th step, she stops at the end of that step; otherwise, she continues with the next step, taking at most <math>3</math> steps in all. What is the probability that Amelia’s position when she stops will be greater than <math>1</math>?
 
 
 
<math>\textbf{(A) }\frac{1}{3} \qquad \textbf{(B) }\frac{1}{2} \qquad \textbf{(C) }\frac{2}{3} \qquad \textbf{(D) }\frac{3}{4} \qquad \textbf{(E) }\frac{5}{6}</math>
 

Latest revision as of 12:58, 4 December 2022